subject
Mathematics, 17.10.2020 19:01 savvaggeb

Fill in the missing statement and a reason in the proof of the corresponding angles theorem "It is given that AB is parallel to CD and points E, G,H, and F are collinear. The measure of EGF is 180°, by the definition of a straight angle. AGE and AGF I adjacent So the measure of AGE plus the measure of AGF equals the measure of EGF by the angle addition postulate. Then substituting for the measure of EGF it can be said that the measure of AGE plus the measure of AGF equals 180°. , so the measure of CHE pulse the measure of AGF. The measure of AGE is equal to the measure of CHE . Finally by the definition of congruence, AGE is congruent to CHE."

ansver
Answers: 3

Another question on Mathematics

question
Mathematics, 21.06.2019 13:20
Charles is on a 8 1/2 mile bike ride. he stops for a rest after he’s gone 5 2/5 miles. how much farther does he still have to go?
Answers: 1
question
Mathematics, 21.06.2019 16:00
Arectangle with an area of 47 m² is dilated by a factor of 7. what is the area of the dilated rectangle? enter your answer in the box. do not leave your answer as a fraction.
Answers: 1
question
Mathematics, 21.06.2019 16:20
Which mathematical statements are true? 1) if 3 is an odd number, then 3 times 3 is an even number. 2) if 6 is less than 7, then 4 is greater than 7. 3) six is divisible by 3, and 10 is a multiple of 2. 4) the average of the data is greater than the largest value in the data, or it’s less than the largest value in the data. 5) the slope of a linear graph is its rate of change, and the graph’s y-intercept is the initial value. 6) if an equilateral triangle has equal angles, then all its angles will measure 45°.
Answers: 3
question
Mathematics, 21.06.2019 21:30
The box plots show the average speeds, in miles per hour, for the race cars in two different races. which correctly describes the consistency of the speeds of the cars in the two races?
Answers: 3
You know the right answer?
Fill in the missing statement and a reason in the proof of the corresponding angles theorem "It is...
Questions
question
Mathematics, 12.02.2021 23:50
question
Mathematics, 12.02.2021 23:50
Questions on the website: 13722367